¿Cómo puedo usar un imán para levantar un clip?

Esta pregunta ya se ha hecho , pero no apareció ninguna respuesta satisfactoria. Todas las respuestas parecen llevar el problema más lejos, pero no explican claramente lo que está pasando.
Lo reformularé para un caso específico, le diré lo que he recopilado de las respuestas y lo que espero específicamente.


Configuración : tengo un potente ferroimán y un clip sobre la mesa. Toco un polo del ferroimán con el clip, luego lo levanto todo. El clip sale con el ferroimán.

Cuando jalo el ferroimán, estoy aplicando una fuerza sobre él, por lo tanto, trabajo sobre él, proporcionando la energía para levantar tanto el imán como el clip hasta el pozo de potencial.


El problema : el clip experimenta la atracción de la gravedad y no la atracción de mi mano, sino la atracción del campo magnético. Me parece que el campo magnético está trabajando en el clip.

Sin embargo, la fuerza de Lorentz causada por un campo magnético sobre una partícula en movimiento siempre es perpendicular al movimiento, por lo que no realiza trabajo. Esto es cierto también para el movimiento restringido. El campo magnético no puede realizar trabajo sobre los átomos del clip .

Aqui estamos. Un clip está hecho de partículas cargadas: electrones y núcleos. Los campos magnéticos no pueden realizar trabajo sobre partículas cargadas. Sin embargo, hay una fuerza de atracción innegable en el clip por el imán, y esta fuerza es capaz de levantarlo en un pozo de potencial, por lo que funciona.


La respuesta que busco:

Me parece que en el paradigma de la mecánica clásica + fuerzas de Lorentz + partículas cargadas puntuales esta pregunta no tiene una respuesta consistente.

Dos opciones:

1) Puede probar que estoy equivocado, mostrándome claramente cómo puede el campo magnético del imán trabajar en un conjunto de partículas.

2) Si tengo razón, entonces me gustaría ver qué otras nociones son necesarias ya sea tratando a los electrones como corrientes y usando ecuaciones de Maxwell o introduciendo espines cuánticos. También quiero ver cómo estos nuevos conceptos implican que el campo de un imán puede realizar un trabajo sobre una pieza de metal.

NB: se le permite tratar tanto el clip como el imán como infinitos en extensión en dos de tres dimensiones espaciales.

El clip es ferromagnético y el campo magnético induce un momento magnético considerable. El campo magnético no es uniforme y, como tal, puede producir una fuerza neta en un momento magnético.
Te escucho @LLlAMnYP, ¿podrías detallar cómo surge un momento magnético a partir de partículas puntuales? Además, si lo que dices sobre los campos magnéticos no uniformes es cierto, ¿significa que si el imán y el clip fueran planos infinitos, no se atraerían?
El momento magnético macroscópico surge de los momentos magnéticos individuales de los electrones. El momento magnético del electrón es una propiedad fundamental del electrón. En los materiales ferromagnéticos, los momentos magnéticos de los electrones se alinean en paralelo dando el momento macroscópico.
Las partículas puntuales tienen un momento magnético intrínseco, más comúnmente conocido como su giro . No hagamos el clip infinito, solo el imán. Entonces sí, no debería haber ninguna fuerza neta. Pero puede probar esto enrollando un solenoide con un núcleo de aire. El clip se sentiría atraído hasta su extremo, pero no llegaría hasta el centro, donde el campo es bastante uniforme.
En nuestro curso de laboratorio de baja temperatura teníamos un imán superconductor que generaba campos de 8T. El solenoide fue enfriado por un refrigerador de ciclo cerrado, pero a través del medio había un orificio de libre acceso. Nuestro tutor nos contó una historia divertida, cómo dejaron que alguien se acercara demasiado con unas pinzas. Esos fueron acelerados hacia el centro del solenoide, pero después de eso experimentaron poca o ninguna fuerza y ​​con el impulso acumulado se clavaron una pulgada en la mesa de madera sobre la que estaba parado el imán.

Respuestas (4)

Lo importante aquí es que un dipolo magnético, como un magnetismo magnético permanente o inducido en material ferroso, produce un campo no uniforme .

La energía potencial de un dipolo magnético. m en un campo magnético B es

tu = m B .
Con mayor frecuencia (como en la respuesta de anna v), esto se usa para explicar el par que hace que el momento magnético quiera alinearse con el campo externo: la energía se minimiza si m y B son paralelos. Supongamos que ya están alineados; encontramos la fuerza como
F = tu .

Sin embargo, el gradiente de un producto escalar tiene una expansión sorprendentemente complicada , que puedes verificar expandiendo todos los términos de los componentes.

( m B ) = ( m ) B + ( B ) m + m × ( × B ) + B × ( × m ) .
Podemos simplificar esto considerando m constante, por lo que esos gradientes desaparecen. De las ecuaciones de Maxwell tenemos × B = 0 . Finalmente, definamos nuestro sistema de coordenadas de modo que m (y por lo tanto B , dado que ya asumimos que están alineados) apuntan a lo largo de la z -eje. Eso nos deja con
F = ( m ) B = ( m z ) B z z ^ .
Entonces para un dipolo permanente m en un campo B encontramos tres casos límite:

  1. Si m , B son paralelos, el dipolo sentirá una fuerza en la dirección creciente | B |

  2. Si m , B son antiparalelos, el dipolo sentirá una fuerza en la dirección decreciente | B |

  3. Si m , B no son paralelos, el dipolo sentirá un torque que lo hará querer alinearse con el campo.

Esta es más o menos mi experiencia con los imanes permanentes. Para hacer que los imanes permanentes se repelan, debe restringir su rotación de alguna manera; lo que les gusta hacer es dar la vuelta y atraer. El magnetismo inducido (por ejemplo, clips) es el resultado de muchos pares de alineación microscópicos.

Puede encontrar el mismo resultado si observa que la energía almacenada en un elemento de volumen del campo magnético es d tu = ( H B ) d 3 X , y encontrar la disposición de los imanes que minimice el volumen del campo fuerte. Esto es relativamente intuitivo para dipolos paralelos alineados de extremo a extremo, que tienen un fuerte campo en el espacio vacío entre los imanes, y también para dipolos antiparalelos colocados de lado a lado, donde se suman los "campos de retorno" entre los dipolos. Sin embargo, para ver los casos repulsivos, debe hacer una integral desordenada sobre los campos marginales para confirmar que las configuraciones de dipolo distante tienen menos energía almacenada que las configuraciones de dipolo cercano pero no superpuesto.

En cuanto al argumento de que la fuerza de Lorentz

F = d pag d t = q metro pag × B
no puede hacer trabajo, porque la fuerza es perpendicular al momento y por lo tanto no puede cambiar la magnitud de | pag | : este argumento asume que el campo B visto por la partícula es uniforme. Si B varía a lo largo de la trayectoria de la partícula, la partícula (en su marco de reposo) ve una variable en el tiempo B y un campo eléctrico que obedece B t = × mi . Es el campo eléctrico inducido el que hace el trabajo. Hay un buen problema en el libro de texto de E&M de Griffiths que funciona a través del argumento.

Me gustaría agregar un suplemento a la respuesta aceptada. La respuesta aceptada es una excelente derivación basada en principios físicos, pero después de leerla, no pude evitar sentir que la respuesta es mucho más abstracta que la pregunta. Esto hace que parezca que el misterio se está moviendo de la pregunta original "¿Cómo pueden funcionar las fuerzas magnéticas?" a "¿Por qué la energía potencial de un dipolo magnético está dada por m B ?" Ahora, por supuesto, es posible abordar la última pregunta de varias maneras (por ejemplo, uno podría tomar esta respuesta para un ciclo actual y luego hacer que el ciclo sea infinitesimalmente pequeño). Sin embargo, quiero abordar este problema desde un punto de vista puramente intuitivo y usar nada más que la fuerza de Lorentz y las ecuaciones de Maxwell.

Antes de comenzar, quiero comentar que múltiples respuestas tanto aquí como en otros lugares han abordado el tema fundamental, que es que un dipolo no es lo mismo que una partícula cargada. (Hablando muy libremente, un dipolo se puede imaginar como una corriente, o múltiples partículas cargadas en movimiento, cuya carga eléctrica neta es cero, y que están atascadas en una determinada configuración. Por lo tanto, se puede atribuir el "trabajo" realizado a cualquier fuerza que sostenga el dipolo juntos, o, según el caso considerado, a las fuerzas que mantienen unido un conjunto de dipolos, o a otras fuerzas específicas de una situación particular). Aunque esto indica claramente el error fundamental en el argumento, todavía no da una explicación concreta. imagen mental de cómo la fuerza de Lorentz conduce al trabajo que se realiza.

Argumento Intuitivo

Para simplificar, imagine que un imán de barra consta de una colección de dipolos magnéticos que son todos paralelos al z eje. Cada uno de estos dipolos magnéticos se puede reemplazar por un bucle de corriente infinitesimal, por lo que es razonable imaginar que todo el imán se reemplaza por un gran bucle de corriente en un plano definido por z = constante . Esto nos da el beneficio de imaginar el imán como una colección de partículas en movimiento, de modo que podamos referirnos a la fuerza de Lorentz. El modelo más simple de un bucle de corriente de este tipo es un cable resistivo conectado a una batería.

En lugar de considerar la interacción entre este bucle de corriente y un clip o una segunda barra magnética, simplifiquemos el problema postulando que existe un campo magnético externo invariable en el tiempo cuya fuente está muy lejos. La pregunta es si este campo magnético puede realizar trabajo en el circuito de corriente.

Considere tres casos:

  1. El campo magnético externo B tiene un componente que no está en el z dirección.

    En este caso, es muy fácil hacer un dibujo y confirmar, usando la ley de Lorentz, que el bucle normalmente experimentará una fuerza neta o un momento de torsión neto y, por lo tanto, se realizará trabajo sobre él. Esto se explora cuantitativamente aquí .

    • Considere el caso donde el lazo es un círculo y B contiene un componente que va radialmente hacia el exterior. Habrá una fuerza neta en el z dirección.

    • Considere el caso donde B es uniforme Habrá un par neto pero no una fuerza neta.

  2. El campo magnético externo B es paralelo a z , pero varía en valor a través del espacio.

    Esta es una violación de las ecuaciones de Maxwell, que requieren B = B z z = 0 . Entonces, podemos ignorar este caso.

  3. El campo magnético externo B es paralelo a z y uniforme. En este caso, no hay fuerza neta ni par neto, por lo que no hay misterio que explicar.

Comentarios adicionales

Hay un par de detalles que escribir el argumento anterior me obligó a considerar.

Más detalles sobre el caso 2

Primero, la interpretación del caso 2 puede ser algo más complicada que lo que escribí anteriormente. Esto se debe a la idealización del alambre como unidimensional. Se podría disponer el cable de modo que, en el cable , B es paralelo a z y tiene magnitud constante, pero B depende de X y y de modo que B = 0 , y B z cambia fuera del cable. (No estoy del todo seguro de si la construcción de tal B que satisface globalmente B = 0 es posible, pero supongo por el momento que lo es).

La fuerza en este caso sería la misma que en el caso 3, es decir,

F = I ( d × B ) = I ( d ) × B = 0 ,
desde B es constante a lo largo del camino de la integración. Un argumento similar muestra que el torque es cero. Entonces, una vez más, no hay misterio que explicar. Tenga en cuenta que, para un objeto con volumen finito, la situación descrita en este párrafo probablemente se clasificaría como parte del caso 1.

Relación con el modelo de un dipolo como un bucle de alambre infinitesimal

Otro punto a considerar es cómo los tipos de argumentos presentados en esta respuesta se relacionan con la expresión dada para un dipolo infinitesimal en la respuesta aceptada. Allí, para un dipolo con m = m z ^ , la fuerza sobre el dipolo viene dada por m z B z z ^ . Sin embargo, en el último párrafo admití que, al menos para un bucle finito, puede ser posible generar un campo magnético donde se cumplan condiciones similares, pero la fuerza es cero.

La clave aquí es darse cuenta de que, para un dipolo infinitesimal, las ecuaciones de Maxwell imponen una restricción muy fuerte en la relación entre z B z y los otros componentes de campo a lo largo del cable. Para simplificar, tome el caso donde el alambre es un lazo de radio a en el avión z = 0 y nos interesará tomar el límite donde a es muy pequeño. En el centro del bucle, B ( 0 ) = B 0 z ^ . eAdemás, suponga que el campo magnético es radialmente simétrico. Usaré coordenadas cilíndricas y suprimiré ϕ (debido a la simetría) y z (porque solo nos interesa el avión z = 0 ). En el bucle, el campo tiene la forma

B ( a ) = ( B 0 + a r B z ) z ^ + a r B r r ^
Todos los derivados se evalúan a 0 . Ahora el z -El componente no contribuirá con una fuerza neta, como se discutió anteriormente, pero observe que puede haber un componente radial, que sabemos que puede causar una fuerza neta en el bucle. Obtenemos
F = d × B = I 0 2 π ( a d ϕ ϕ ^ ) × ( a r B r r ^ ) = 2 π a 2 I r B r z ^ = 2 m r B r z ^
La observación crítica es que B = 0 implica que r B r = 1 2 z B z . Entonces la expresión anterior se convierte en
F = m z B z z ^
que se derivó anteriormente.

El punto es que las ecuaciones de Maxwell requieren que no puedas tener un valor distinto de cero. z B z sin tener también una derivada distinta de cero para otras componentes del campo magnético, y ya hemos visto cómo las otras componentes pueden causar una fuerza en el z dirección.

Es un hecho experimental que los átomos y las moléculas tienen momentos magnéticos. También las partículas elementales con espín tienen momentos magnéticos , así como protones y neutrones.

No es la mecánica clásica la que describirá el comportamiento de los átomos y las moléculas, sino la mecánica cuántica.

Existen modelos teóricos mecánicos cuánticos del magnetismo , y al principio de la historia de la física para el ferromagnetismo .

ferromodelo

Así que no son conceptos realmente nuevos, sino tan antiguos como la mecánica cuántica.

Editar para responder a:

Un clip está hecho de partículas cargadas: electrones y núcleos. Los campos magnéticos no pueden realizar trabajo sobre partículas cargadas.

Pero pueden trabajar en momentos dipolares magnéticos.

Sin embargo, hay una fuerza de atracción innegable en el clip por el imán, y esta fuerza es capaz de levantarlo en un pozo de potencial, por lo que funciona.

Un potencial magnético se puede definir macroscópicamente.

Un momento dipolar magnético en un campo magnético poseerá energía potencial que depende de su orientación con respecto al campo magnético. Dado que las fuentes magnéticas son inherentemente fuentes dipolares que se pueden visualizar como un bucle de corriente con corriente I y área A, la energía se suele expresar en términos del momento dipolar magnético:

fórmula

La energía se expresa como un producto escalar e implica que la energía es más baja cuando el momento magnético está alineado con el campo magnético. La diferencia de energía entre alineado y anti-alineado es

energía potencial del momento magnético

La expresión de la energía potencial magnética se puede desarrollar a partir de la expresión del par magnético en un bucle de corriente.

Estas relaciones para un bucle de corriente finito se extienden a los dipolos magnéticos de las órbitas de los electrones ya los momentos magnéticos intrínsecos asociados con el espín del electrón y el espín nuclear.

Sin embargo, hay una fuerza de atracción innegable en el clip por el imán, y esta fuerza es capaz de levantarlo en un pozo de potencial, por lo que funciona.

Habiendo aceptado que las partículas elementales son dipolos permanentes (que, por supuesto, es QM), no es realmente necesario invocar QM para explicar la presencia de fuerzas netas (no solo pares) que actúan sobre los dipolos. Pero siento que la respuesta a esta pregunta realmente carece de una explicación de lo que le sucede al sistema cuando dos dipolos se atraen entre sí. He intentado calcular la integral de H 2 cuando dos dipolos están separados y cuando están juntos, para ver si la energía del campo se reduce cuando se mueven juntos, pero tienen que irse ahora y no tuvieron tiempo de continuar.
@LLlAMnYP ve clásicamente este hyperphysics.phy-astr.gsu.edu/hbase/magnetic/magpot.html . . Sin embargo, en realidad no funciona a nivel atómico.
Oh, esto es lo que he visto. Estaba pensando en algo más fundamental. Por ejemplo, si un dipolo es un bucle de alambre superconductor con corriente, a medida que entra en la región de campo más fuerte, la corriente que fluye a través de él disminuiría. Pero estoy tratando de encontrar algún proceso similar que funcione para, digamos, dos imanes de barra cuando se juntan.

Es cierto que la fuerza de Lorentz no puede realizar ningún trabajo. Como han señalado otros, un campo magnético no uniforme puede trabajar en dipolos intrínsecos. Siento que también se debe señalar algo más: el campo magnético puede realizar un trabajo sobre el campo eléctrico, que luego realiza un trabajo sobre las partículas cargadas. Ver: el fenómeno de la inducción magnética donde un campo magnético cambiante produce un campo eléctrico que realiza un trabajo.